Banking & Finance

1

Consider the following statements regarding the measures of supply of money:

  1. M1 and M2 are called narrow money
  2. M3 and M4 are called broad money
  3. M1 is the least liquid

Which of the above statements is/are true?

A
Only 1
B
1, 2
C
2, 3
D
1, 2, 3

2

Consider the following statements:

  1. Under Cash Reserve Ratio, banks have to keep a certain part of their deposits with themselves
  2. Under Statutory Liquid Ratio, banks have to hold a part of their total demand and term deposits with the RBI

Which of the above statements is/are true?

A
Only 1
B
Only 2
C
Both 1 and 2
D
Neither 1, nor 2

3

The Reserve Bank of India takes which of the following measures to maintain profitable Reserve Deposit Ratio (RDR)?

  1. Cash Reserve Ratio
  2. Statutory Liquid Ratio
  3. Bank Rate

Choose the correct answer using the codes given below:

A
Only 1
B
1, 2
C
1, 3
D
1, 2, 3

4

Consider the following statements:

  1. RBI buys and sells shares under the ‘Open market transaction’
  2. Bank Rate is the interest rate at which a commercial bank gets loan from RBI

Which of the above statements is/are true?

A
Only 1
B
Only 2
C
Both 1 and 2
D
Neither 1 nor 2

5

Which of the following is/are the part/s of RBI’s Quantitative Restriction instruments?

  1. Bank Rate
  2. Statutory Liquidity
  3. Marginal requirements
  4. Open market transactions

Choose the correct answer using the codes given below:

A
1, 2
B
1, 2, 4
C
2, 3, 4
D
1, 2, 3, 4

6

Which of the following statements regarding Indian banks is false?

A
Indian Banks have to inform the Reserve Bank of India from time to time about how much loan they are giving to which sectors and at what interest rates
B
There is no institution to track the activity of loan givers of the formal sector
C
The Reserve Bank of India monitors the functioning of informal sources of loans
D
Informal sector charges high rate of interest because there are no rules and regulations in informal sector

7

Which of the following is the main source of Bank’s Income?

A
Cash deposited by the depositors
B
Interests paid by the borrowers
C
Difference between interest paid to the depositors and interest paid by the borrowers
D
All of the above

8

Which of the following statements is true with respect to Reserve Deposit Ratio?

A
The fraction of their total deposits which commercial banks keep as reserves
B
The RBI Determines the Reserve Deposit Ratio
C
Both (a) and (b)
D
None of the above

9

To maintain Reserve Deposit Ratio with commercial banks, the RBI uses many policy measures and instruments. Which of the following is/are such instruments?

  1. Cash Reserve Ratio
  2. Statutory Liquidity Ratio
  3. Bank Rate

Choose the correct answer using the codes given below:

A
Only 1
B
Only 2
C
1 and 3
D
All of the above

10

Which of the following statements is/are true?

  1. The total amount of currency stock in the economy is much higher than the total amount of high powered money
  2. Commercial banks create this additional amount of currency by providing a portion of their deposit in the form of loans or investment credentials

Choose the correct answer using the codes given below:

A
Only 1
B
Only 2
C
Both of the above
D
None of the above

11

Which of the following is the condition of Bank Run?

A
A situation where everybody wants to take money out of one’s bank account before the bank runs out of reserves
B
Highest level of profit of banks
C
Giving lot of responsibilities to a bank
D
More than one options of the above

12

Consider the following statements regarding the RBI:

  1. The most important role of the RBI is to supply currency and control monitory policy
  2. RBI is an independent body for monetary policy operation
  3. The RBI increases or reduces the flow of High Powered Money in the economy

Which of the above statements is/are true?

A
1, 2
B
2, 3
C
1, 3
D
All of the above

13

Which of the following is/are part of the ‘reserve money’ in India?

  1. Net credit given by RBI to Banks and Government.
  2. Government bonds held by RBI.
  3. Forex reserve available with RBI.

Choose the correct code(s):

A
I and III
B
II and III
C
III Only
D
I and III

14

The Securities and Exchange Board of India has been set up to:

  1. Protect the interests of investors.
  2. To regulate the activities of brokers in the stock market.
  3. To ensure transparency in operation in the stock market.
  4. To encourage a healthy growth of the market.

Which of the statement(s) given above is/are correct?

A
I and II
B
I, II and IV
C
I, II and III
D
I, II, III and IV

15

Consider the following statements about Real time Gross settlement (RTGS) system.

  1. It operates on an order by order basis settlement system.
  2. It is primarily meant for large value transactions.
  3. RTGS system is also known as National Electronic Fund Transfer (NEFT) system.

Which of the statement(s) given above is/are TRUE?

A
I only
B
I and II
C
I and III
D
All of the above

16

What is SLR?

A
The ratio of laquid assets to net demand and time liablities (NDTL).
B
The share of net demand and time liabilities that banks must maintain as cash balance with the Reserve Bank.
C
The purchase/sale of government securities by public and market
D
The ratio of rediscounted bills of exchange and other government securities in the market.

17

Functions of the commercial bank includes

  1. Accepting deposits
  2. Providing loan
  3. Building credit
  4. Determining Cash Reserve Ratio (CRR)

Codes:

A
1 and 2
B
1, 2 and 3
C
2, 3 and 4
D
All of the above

18

The banks invest their surplus funds in some securities. What constitutes these securities?

  1. Cash
  2. Gold
  3. Treasury bills and National Savings Certificates
  4. Securities of state sponsored bodies like electricity boards, housing boards, debentures of Land Development Banks, shares of RRBs etc.
A
1 & 2 only
B
1, 2 & 3 only
C
3 & 4 only
D
All of the above

19

Which of the following terms indicates a mechanism used by commercial banks for providing credit to the Government?

A
Cash credit ratio.
B
Debit service Obligation.
C
Liquidity adjustment facility.
D
Statutory liquid ratio.

20

NPCI (National Payment Corporation of India) is usually in news and is the umbrella organisation for the retail payment. NPCIL is an initiative of:

A
RBI only
B
Ministry of Finance and RBI
C
RBI and SBI
D
RBI and Indian Banks’ Association

21

Recently, Banks like SBI, ICICI have increased the Marginal Cost of Funds Based Lending Rate (MCLR). Consider the following statements with respect to MCLR:

  1. This is the minimum rate below which banks can’t lend to anyone.
  2. This has replaced the Bank rate from April 2016.

Choose the correct code

A
Only 1
B
Only 2
C
Both 1 & 2
D
Neither 1 nor 2

22

Which of the following is not an All India Financial Institution regulated by RBI?

A
NABARD
B
IFCI
C
EXIM
D
NHB

23

Consider the following statements about Financial Stability Development Council (FSDC):

  1. FSDC is chaired by Finance Minister and Vice Chairman of Niti Aayog is its Deputy Chairman.
  2. FSDC Sub Committee is headed by Chairman of Insolvency and Bankruptcy Board of India.
  3. FSDC is a statutory body that looks into the overall financial stability of the India’s economy.

Choose the correct code

A
2 Only
B
3 Only
C
1, 2 and 3
D
None of the above

24

Consider the following statements:

  1. The repo rate is the rate at which the RBI borrows money from commercial banks.
  2. Reverse Repo rate is the rate at which other banks borrow from the Reserve Bank of India.
  3. In India, RBI uses repo and reverse repo techniques to increase or decrease the liquidity in the market.
  4. Repo and reverse rates techniques can also be used to manage liquidity if there is an excessive capital flow from abroad.

Which of the statements given above is/are incorrect?

A
1 and 2
B
3 only
C
1, 2 and 4
D
1, 2 and 3

25

Consider the following statements:

  1. Lower the currency deposit ratio, higher the money multiplier.
  2. Lower the reserve deposit ratio, higher the money multiplier.
  3. Larger the monetary base, higher is the money supply.
  4. Larger the monetary base, lower is the money supply.

Which of the above statement(s) is/ are correct?

A
1 and 3 only
B
1, 2 and 4 only
C
1, 2 and 3 only
D
2 and 4 only

26

Consider the following statements about the Cooperative Banks in India?

  1. They are required to assist the weaker sections and agricultural sector.
  2. They are organized under the state government’s laws.
  3. They have access to liberal financial assistance from NABARD
  4. They are organized in a tiered structure. Of these statements
A
I, III and IV are correct
B
I and IV are correct
C
I and III are correct
D
All are correct

27

Which one of the following is the most important determinant of the price of shares of a company?

A
Past track record of the company
B
Current performance of the company
C
Perception about its future performance
D
Macro economic situation of the country

28

A derivative security issued against the backing of a homogeneous category of mortgaged assets is called:

A
Mortgage Backed Securities
B
Collateralized Debt Swaps
C
Collateralized Debt Obligations
D
Collateral Mortgage Securities

29

NRE deposit is:

A
Non Resident External deposit
B
Non Resident Exchange deposit
C
Non Refundable External deposit
D
Non Resident Extended deposit

30

Consider the following statements about Real time Gross settlement (RTGS) system.

  1. It operates on an order by order basis settlement system.
  2. It is primarily meant for large value transactions.
  3. RTGS system is also known as National Electronic Fund Transfer (NEFT) system.

Which of the statement(s) given above is/are TRUE?

A
I only
B
I and II
C
I and III
D
All of the above

31

Match the following:

List A (term) List B (definition)
A. Stagflation 1. Increase in the general price level of goods and services.
B. Depression 2. Decline in the Gross Domestic Product (GDP) for two or more consecutive quarters.
C. Inflation 3. Inefficient economic productivity, high unemployment and falling price levels.
D. Recession 4. Slow economic growth and relatively high unemployment.
A
A-1; B-2; C-3; D-4
B
A-4; B-3; C-1; D-2
C
A-1; B-2; C-4; D-3
D
A-4; B-3; C-2; D-1

32

Consider the following statements-

  1. If RBI increases the Repo Rate then the inflation in the market will increase.
  2. Inflation is generally caused by the excess supply of money in the market and subsequently reduction in the supply of commodity.

Which of the statement given above is/are correct?

A
I Only
B
II Only
C
Both
D
None

33

Consider the following about Brown Label ATMs

  1. They are owned by the bank while some key functions are managed by third party.
  2. They carry the logo of bank.

Choose the correct code(s)

A
I Only
B
II Only
C
Both I and II
D
Neither I nor II

34

Misery Index is defined as:

  1. Inflation plus rate of unemployment.
  2. Inflation plus rate of unemployment plus rate of interest.
  3. Inflation as measured by consumer Price Index only.
  4. Inflation as measured by Wholesale Price Index only.
A
Only (1) and (2)
B
Only (2) and (4)
C
Only (1) and (3)
D
Only (1) and (4)

35

What is/are most likely the reasons for growing NPAs in the banks?

  1. Global and domestic slowdown
  2. Delayed clearances of various projects
  3. Deteriorating asset quality
  4. Economic policy of the government and politician-corporate nexus.

Choose the correct option:

A
1 and 2
B
3 only
C
2, 3 and 4
D
1, 2 and 4

36

Consider the following statements.

  1. Core inflation is a measure of the total inflation which is affected by areas of market which may experience sudden inflation spiky such as food or energy.
  2. Underlying inflation while measuring inflation excludes highly volatile products such as food and energy prices.

Which of the statement(s) given above is/are TRUE?

A
Only I
B
Only II
C
Both I and II
D
Neither I nor II

37

With which of the following the term ‘money illusion’ is associated with?

A
Capital Market
B
Inflation
C
Banking
D
Money market

38

What are the functions of RBI?

  1. Custodian of Cash Reserves of Commercial Banks
  2. Central Clearance and Accounts Settlement
  3. Controller of Credit
  4. Government's Banker
  5. Banker's Bank

Choose the correct option:

A
2, 3 and 4
B
1, 3 and 5
C
1, 3, 4 and 5
D
All of the Above

39

Consider the following statements-

  1. Inflation causes by excess supply of money in the market.
  2. To curtail the impact of inflation RBI raises Repo rate, Reverse Repo rate and Bank rate to control the flow of money in the market.

Which of the statement given above is /are correct?

A
Both
B
I only
C
II only
D
None

40

Consider the following definitions of the terms associated with inflation:

  1. Stagflation is a slowdown in the rate of increase of the general price level of goods and services in a nation’s gross domestic product over time.
  2. Recession is a situation in which the inflation rate is high and the economic growth rate is low.
  3. Reflation is the act of stimulating the economy by increasing the money supply or by reducing taxes.

Which of the above statements is/are not correct?

A
Only I
B
Only II
C
Only III
D
I and III

41

Depreciation of rupee has overall effect on the economy. which of the following can be considered as major impact of rupee depreciation on Indian economy?

  1. Imports become costlier due to depreciation of rupee.
  2. Overseas bank loans will become costlier to repay.
  3. Current account deficit will widen.

Which of the above are correct?

A
I and III
B
I and II
C
II and III
D
all

42

Which of the following cannot be considered as a major reason for inflation in Indian economy?

A
Demand and supply imbalances in various sectors.
B
Imported inflation i.e. fluctuation in prices of commodities such as crude oil and food in international market.
C
Decrease in administered prices of diesel.
D
Speculation in future market trading.

43

Consider the following statements about inflation:

  1. Inflation is a sustained increase in the general price level of goods and services in an economy over a period of time.
  2. Money supply growing faster than the rate of economic growth causes a long sustained period of inflation.
  3. The inflation rate is calculated in India by calculating the movement or change in the wholesale price Index.

Which of the above statements is/are correct?

A
I and II only
B
I only
C
I and III only
D
all.

44

Consider the following statements about Stagflation and select the correct answer:

  1. Stagflation is a condition of slow economic growth and relatively high unemployment accompanied by a rise in prices, or inflation.
  2. Stagflation can result from inappropriate macroeconomic policies.
A
I only
B
II only
C
Both
D
None.

45

Consider the following statements about Inflation Indexed Bonds. Which of the following is/are true?

  1. The bonds will be indexed against retail price index.
  2. Investing in inflation-indexed bonds will have tax benefits.
  3. The income under the bonds would be taxable as in the case of any other government security.

Codes:

A
I, II only
B
II, III only
C
I, III only
D
III only.

46

Which of the following are the reasons for demand-pull inflation?

  1. Increase in forex reserve
  2. Deficit financing
  3. Increase in budgetary deficit
  4. Increase in population

Select the answer from the codes given below:

A
I, II and IV
B
II and IV
C
II, III and IV
D
All of these

47

Consider the following statements:

  1. Rise in Inflation reflects loss of real value in the medium of exchange and unit of account within the economy.
  2. Inflation leads to increase in the opportunity cost of holding money.
  3. If the inflation rate is greater than that of other countries, domestic products become less competitive.

Select the correct answer using the codes given below:

A
1 and 2 only
B
2 and 3 only
C
1 and 3 only
D
All of the above

48

Suppose inflation over the next year is expected to be 5%, and assume there are no supply shocks. What rate of inflation will the short-run Phillips curve show at the natural rate of unemployment?

A
0%
B
Between 0% and 5%
C
5%
D
Over 5%

49

Suppose the Reserve Bank of India wishes to restrain inflation, so it increases Bank Rate. Which of the following events will not be part of the monetary policy transmission process?

A
The increased Bank Rate will increase the value of sterling.
B
The increased Bank Rate will increase people’s wealth.
C
The increased Bank Rate will lead to increases in other nominal interest rates.
D
The increased Bank Rate will reduce the incomes people expect to have in future.

50

What is inflation targeting?

A
It is the economic policy where the government targets a specific rate beyond which the inflation should not proceed.
B
It is the economic policy where by the central bank targets a specific rate of inflation and then steers the actual rate of inflation towards it by means of monetary policy instrument.
C
It is the economic policy where by both the central bank and central government targets a specific rate of inflation beyond which it should not proceed.
D
It is the economic policy whereby the market forces are allowed to target the inflation rate without any interference of the central bank or government.

51

Which of the following are the cost-push causes of inflation?

  1. Deficit financing
  2. Increase in administered prices
  3. Increase in interest rate
  4. Increase in population
  5. Increase in oil prices
  6. Mounting public expenditure

Select the correct answer from the codes given below:

A
II, III, V and VI
B
II, III and V only
C
III and V only
D
I, II, IV and VI

52

Consider the following statements about Inflation Index Bond:

  1. They are directly sold by Reserve Bank of India.
  2. They provide new investment avenue with positive real interest rate in period of inflation.
  3. Capital gain tax is not applicable for them.

Which of the above statement(s) is/are incorrect?

A
II and III
B
I and III only
C
I only
D
III only

53

Repo and Reverse Repo rate form a part of liquidity adjustment facility used by RBI to control inflation. In this context consider the following statements if RBI decreases Repo rate then

  1. It act as incentive for banks to lend more money
  2. It acts as disincentive for banks to lend more money
  3. There will be increase in money supply in economy
  4. There will be decrease in money supply in economy

Which of the following statement(s) is/are correct?

A
I and III
B
II and IV
C
I and IV
D
II and III

54

Which macroeconomic variables in an economy are affected by changes effected through fiscal policy?

  1. Aggregate demand and the level of economic activity.
  2. Savings and Investment in the economy.
  3. The distribution of income.

Choose the correct option:

A
1 only
B
1 and 2
C
2 and 3
D
All of the Above

55

Inflation is caused as a result of:

A
Increase in money supply.
B
Fall in production.
C
Increase in money supply without a corresponding increase in production.
D
Decrease in money supply without a corresponding decrease in production.

56

Stagflation refers to a situation of :

A
High inflation, low unemployment.
B
High inflation, high unemployment.
C
Low inflation, high unemployment.
D
High inflation, low unemployment.

57

What do you mean by stagflation?

A
When economic growth is slow but there is high rate of price inflation.
B
Inflation remain between 3-10% a year, but heats up economic growth too fast.
C
Money loses value so fast that business and employee income can't keep up with costs and prices
D
When an asset bubble bursts, and prices fall and it is harder to stop than inflation.

58

Which of the following is/are true for base rate?

  1. It is the minimum interest rate of a bank below which it is not viable to lend.
  2. All categories of domestic rupee loans are priced only with reference to the Base Rate.
  3. It leads to more transparency but hampers monetary policy transmission.
Choose the correct option:
A
1 and 2
B
1 only
C
All of the above
D
2 and 3

59

What are the tools used by RBI to control inflation?

  1. Preventing hoarding and cartelization by rationing of credit, thus addressing supply side constraints.
  2. Increase in Cash Reserve Ratio.
  3. Increase in repo rates.
  4. Reduction in Bank rate.
  5. Floating Market Stabilization Bonds in market.

Choose the correct statement/s:

A
1, 2 and 4
B
2, 3 and 5
C
1, 3 and 4
D
2, 4 and 5

60

Which factors are responsible for the sluggish growth of bank credit?

  1. incomplete transmission of the monetary policy as banks have not passed on the entire benefit to borrowers
  2. unwillingness of the banks to lend credit on account of rising Nonperforming Assets
  3. worsening of corporate balance sheets, forcing them to put their investment decisions on hold
  4. interest rates in the bond market being more attractive to borrowers

Code:

A
1 and 2
B
2 and 3
C
1, 2 and 3
D
All of the above

61

Paper currency is the main currency of the country consider these statements:

  1. RBI has the sole right to issue all currency notes in the country.
  2. The system governing the note issue in India is Indian Coinage Act.
  3. Paper currency is inconvertible.

Which statement(s) is/are correct?

A
1 only
B
1 & 2 only
C
3 only
D
All of the above

62

A: Headline inflation, based on the Consumer Price Index (CPI) (combined for rural and urban areas) series, dipped during April-January 2015-16.

R: For various subgroups of the CPI (combined), the decline in inflation was broad based and mainly driven by lower inflation of food articles and items under the non-food nonfuel category.

Code:

A
A is true and R is the correct explanation of the A
B
A is true and R is true but R is not the correct explanation of the A
C
A is true but R is false
D
Both A and R are false

63

Which of the following are instruments of qualitative credit control employed by the Central bank?

  1. The margin requirements on secured loans
  2. Persuasion exercised through discussions, letters, speeches, and hints to the banks
  3. Measures to channel credit to particular sectors.
A
1 only
B
2 & 3 only
C
3 only
D
All of the above

64

Which of the following is/are true for Headline inflation?

  1. It reflects the rate of change in prices of all goods and services in an economy over a period of time.
  2. In India, headline inflation is measured through the WPI
  3. It excludes transitory or temporary price volatility as in the case of some commodities such as food items, energy products etc

Select the correct code:

A
1 only
B
1 and 2
C
2 and 3
D
All of the above

65
Commercial banks build currency through
A
Time deposit
B
Demand deposit
C
Treasury Bill
D
Exchange Bill

66

Inflation is the general increase in prices and fall in the purchasing value of money. Which of the following are the reasons for inflation in the economy?

  1. Rising imported raw materials costs
  2. Higher indirect taxes imposed by the Government.
  3. Increase in non-plan expenditure of Government.
  4. Depletion of natural resources.

Select the correct code:

A
1 and 3 Only
B
2 and 4 Only
C
1, 2 and 3 Only
D
All of the above

67

According to the statutory liquidity ratio, the commercial bank should get its liquid assets from:

A
Cash Reserve
B
Gold Reserve
C
Approved securities
D
All of the above

68

Which of the following is/are true for Cash Credit?

  1. The credit limit is determined by the bank's estimation of the borrower's creditworthiness.
  2. The utilization of credit by the customer depends upon his withdrawing power.
  3. The borrower has to pay the interest on the total amount of credit sanctioned.
A
1 only
B
1 & 2 only
C
All of the above
D
None of the above

69

Choose the correct statement:

  1. Consumer Price Index(CPI)is released quarterly by Central Statistical Office(CSO)
  2. Wholesale Price Index (WPI) is also called as Headline Inflation.
  3. WPI caters to commodity inflation better.

Select the correct code.

A
1 & 2 Only
B
3 Only
C
1 & 3 Only
D
None of these

70

If inflation remains high in an economy, which of the following situations can arise?

A
Inflation can drag down growth.
B
Inflation may discourage savings in banks.
C
It will redistribute income from those on fixed savings to those on inflation linked salaries.
D
All of the above.

71

Which of the following is true about inflation?

  1. Inflation is a measure of rise in the general price level or equivalently average level of prices of all the goods and services in an economy.
  2. Inflation is often viewed as a reduction in the purchasing power of money. Inflation does not necessarily mean that the prices of all the goods and services in an economy rise.
  3. In an inflationary situation, the average level of prices rise with some items showing excessive rise while others may not follow the suit.
  4. A macro-economic policy measure mostly used by the countries across the world, to alleviate the adverse effects of inflation is indexation.

Codes:

A
Only (1) and (2)
B
Only (2) and (4)
C
Only (1) and (3)
D
All the above

72

In an imaginary situation where India has open trade only with the U.S. Which of the following conditions would NOT lead to depreciation in the value of Rupee vis-a-vis Dollar?

A
India having higher inflation than the U.S.
B
Indians investing into U.S. in large amounts.
C
Investors pulling back their money from India.
D
A huge inflow of U.S. tourists into India.

73

In order to control inflation, foreign inflow needs to be sterilized. Here sterilization is meant for:

A
Conformity with import- export regulations.
B
Withdrawing equivalent local currency to maintain a desirable rate of exchange.
C
Injecting equivalent local currency in the economy to maintain a desirable rate of exchange.
D
Curbing counterfeit currency from entering into circulation.

74

What do you understand by a GDP Deflator?

  1. It is a comprehensive measure of inflation.
  2. It is a less comprehensive measure of inflation than price indices.
  3. It is available on a quarterly basis.
  4. It is used to convert data compiled over a period into prices prevailing at an earlier point in time.

Choose the correct option:

A
1 only
B
1 and 2
C
1, 3 and 4
D
1, 2 and 3

75

Which of the following statements is False?

A
Net Primary Deficit is Net Fiscal Deficit minus net interest payments.
B
Net interest payment is interest paid minus interest receipt.
C
Gross Primary Deficit is Gross Fiscal Deficit less depreciaton.
D
None of the above

76

What is private placement of securities?

A
Selling securities through open market as a way of raising capital.
B
The sale of securities to a relatively small number of select investors.
C
The process by which investment bankers raise investment capital from large number of investors.
D
Raising capital through issue of shares offered at a special price by a company to its preferred shareholders.

77

What does the effectiveness of a bank rate policy depend upon?

  1. The degree of bank's dependence on borrowed reserves
  2. The extent to which other rates of interest in the market change.
  3. The state of supply of and demand for funds from other sources.
A
1 only
B
1 & 2 only
C
2 & 3 only
D
All of the above

78

Which of the following is/are true for Open Market Operations?

  1. It does not matter whether the securities are brought from or sold to the public or banks.
  2. In developed countries like US, banks are not affected by the OMO.
A
1 only
B
2 only
C
Both
D
None

79

Consider the following statements:

  1. Increased supply of currency is always good for the health of the economy
  2. If goods and services produced by an economy remain unchanged, then supply of additional currency causes inflation

Which of the above statements is/are true?

A
Only 1
B
Only 2
C
Both of them
D
None of them

80

What are the owned funds of the banks?

  1. Paid-up capital
  2. Capital and reserves
  3. Borrowings from RBI
  4. Capital raised through public, financial institutions and OMOs
A
1 only
B
1 & 2 only
C
1, 3 & 4 only
D
All of the above

81

Which of the following statements is/are true regarding the Currency?

A
There is no single currency at international level, which is issued by a Central Authority
B
Unless the purchasing power of a currency is believed to be stable, it cannot be used as an international medium of exchange
C
International currency system has been established to bring stability in international currency operations
D
All of the above

82

How does the Central bank determine the quantity of reserves with itself?

  1. Central bank lending to the other banks.
  2. Open Market Operations.
  3. Marginal Standing Facility.
  4. CRR and SLR.
A
1 & 2 only
B
1 & 4 only
C
2, 3 & 4 only
D
All of the above

83

What are the effects of inflation on following entities?

  1. Salaried people and wage earners with fixed income lose.
  2. Borrowers lose and creditors gain.
  3. Bond and debenture holders lose.
  4. Profit-earners, speculators and black marketers gain

Choose the correct option:

A
1 and 4
B
1, 3 and 4
C
All of the Above
D
None of the above

84

How does a budget impact the economy?

  1. Fiscal discipline.
  2. Proper microeconomic performance.
  3. Allocation of resources.
  4. Provision of programmes and delivery of services.
A
1 & 3 only
B
1, 3 & 4 only
C
2 & 3 only
D
All of the above

85

Which of the following is/are true for Official Reserve Transaction?

  1. They are entered into with some independent motive by two private parties.
  2. They are carried out by the government and the central banks in pursuit of some international economic policy objective.
A
1 only
B
2 only
C
Both
D
None

86

Growth of rural economy depends on availability of adequate financing and credit. What have been the impediments in proper functioning of rural banking?

  1. Chronic underperformance of formal credit institutions
  2. High incidence of overdue installments by the farmers.
  3. Barring commercial banks, other formal institutions have failed to develop a culture of deposit mobilization
A
1 only
B
1& 2 only
C
All of the above
D
2 & 3 only

87

Which factor is considered for comparing countries in terms of development of the country?

A
Economic growth
B
Income
C
Human development
D
Technology

88

What constitutes terms of credit?

  1. Interest rate
  2. Collateral
  3. documentation requirement
  4. mode of repayment
A
1 & 2 only
B
1, 2 & 3 only
C
2 & 3 only
D
All of the above

89

In which of the following places are currency printing presses found in India?

  1. Dewas
  2. Nasik
  3. Mysore
  4. Salboni

Select the correct code:

A
1 and 2
B
2 and 3
C
1, 3 and 4
D
All of the above

90

Which of the given factors can have a major impact on inflation in the country?

  1. Lower oil prices
  2. Slowing Chinese economy
  3. Monsoons
  4. Large pay awards

Code:

A
1, 2 and 3
B
1, 3 and 4
C
2, 3 and 4
D
1, 2 and 4

91

Who issues the coins in India?

A
RBI
B
Government of India
C
NABARD
D
Both a and b

92

Consider the given statements:

  1. Disinflation is a slowing in the rate of price inflation.
  2. Deflation is a general decline in prices, often caused by a reduction in the supply of money or credit.

Which of the above is/are true?

A
1 only
B
2 only
C
Both 1 & 2
D
None

93

Which of the following are time deposits?

  1. Fixed deposits
  2. Balance in saving accounts
  3. Current account deposits
  4. Loans and advances

Select the correct code:

A
1 only
B
2 and 3
C
1, 2 and 4
D
2, 3 and 4

94

How is bank rate different from repo rate?

  1. It usually deals with loans.
  2. No collateral is involved in a bank rate.
  3. Repo rate is generally comparatively lower than a bank rate.
  4. It is usually used to cater the short term fund requirements of businesses.
  5. It usually deals with securities.

Select the correct code:

A
1 and 2
B
1, 2 and 3
C
3, 4 and 5
D
1, 2, 3 and 5

95

Which of the following are the cost push factors of inflation?

  1. Increase in income of workers i.e. salary increase
  2. High Fiscal Deficit
  3. Uncertain weather conditions
  4. Increase in population

Select the correct code

A
1, 2 and 4 Only
B
1, 3 and 4 Only
C
2, 3 and 4 Only
D
1, 2, 3 Only

96

Which of the following does not constitute lending by banks?

A
Cash credit
B
Deposits with fixed period to maturity
C
Short term advances
D
Bank's investments in government securities and other approved bonds

97

Suppose a central bank is concerned about inflation and wishes to raise interest rates. Which of the following instruments would it be inappropriate to use?

A
Interest rate control.
B
Open market operations.
C
Quantitative easing.
D
Reserve ratio controls.

98
What is high powered money?
A
The total liability of the monetary authority of the country.
B
Deposits refundable by RBI on demand deposit.
C
The currency exchangeable at RBI.
D
Total deposits held by government of India.

99

How can we increase money supply in Indian economy?

  1. Decreasing CRR
  2. Increasing SLR
  3. Selling securities through OMO
  4. Market sterilization

Select the correct code:

A
1 only
B
1 and 2
C
2, 3 and 4
D
All of the above

100

The effect of decrease in the interest rate in an economy is:

A
Decrease the consumption expenditure in the economy.
B
Increase the tax revenue.
C
Increase the investment expenditure in the economy.
D
Increase the total savings in the economy.

101

BASEL norms often in news are related to

A
Regulation of inter-country money laundering
B
OECD regulations for prevention of black money
C
Financial discipline in Banking sector
D
Emission standard norms

102

Consider the given statements:

  1. Quantitative Easing refers to a gradual reduction in the monthly purchase of assets by the Central Bank of USA.
  2. Federal Tapering is mainly an asset purchase or asset swap policy to increase money supply to the banks.

Which of the above is/are correct?

A
1 only
B
2 only
C
Both 1 & 2
D
None

103

Which of the following is not an instrument of Selective Credit Control?

A
Regulation of consumer credit
B
Rationing of credit
C
Margin requirements
D
Cash Reserve Ratio

104

Which of the following is/are true for small finance banks?

  1. They can lend money to people.
  2. They will provide banking products small and marginal farmers, micro and small industries, and other organized sector entities.
  3. At least 50% of its loans should constitute loans and advances of up to 25 lakh.
  4. They can undertake financial services like distribution of mutual fund units, insurance products, pension products, and so on.
Select the correct code:
A
1 and 2
B
3 and 4
C
1, 2 and 4
D
All of the above

105

What is/are the scope of activities of Business Correspondents?

  1. No cash transaction/cash handling is permissible for BCs.
  2. Collection of small value deposits.
  3. They are responsible for ensuring KYC and AML compliance.
  4. Aadhaar seeding.

Select the correct code:

A
1 and 3
B
2 and 4
C
1, 3 and 4
D
All of the above

106

Consider the following statements regarding Systemically Important Banks (SIBs):

  1. Identification of global SIBs is done by Basel Committee on Banking Supervision.
  2. SBI and ICICI are considered global SIBs in India.

Which of the above statements is/are correct?

A
1 only
B
2 only
C
Both 1 and 2
D
Neither 1 nor 2

107

Which of the following is/are Indian credit rating agency/agencies?

  1. CARE
  2. ONICRA
  3. ICRA
  4. S & P

Select the correct code:

A
1 and 2
B
1 and 3
C
1, 2 and 3
D
2, 3 and 4

108

What is Unified Payment Interface?

  1. It envisages a payments architecture that is directly linked to achieving the goals of universal electronic payments, a less-cash society.
  2. It is implemented by largest public sector bank SBI.
  3. A UPI-enabled platform bank account can transfer up to Rs.1 lakh instantaneously.

Choose the correct option:

A
1 and 2 Only
B
1 and 3 Only
C
2 Only
D
All of the above

109

NPCI (National Payment Corporation of India) is usually in news and is the umbrella organisation for the retail payment. NPCIL is an initiative of :

A
RBI only
B
Ministry of Finance and RBI
C
RBI and SBI
D
RBI and Indian Banks’ Association

110

What is fiat money?

A
Currency notes
B
Coins
C
Gold or silver coin
D
Both (a) and (b)

111

Which of the following is/are true for Cooperative Banking in India?

  1. Primary Cooperative Agriculture and Rural Development Banks (PCARDBS) operate at district/block level.
  2. They are not subject to CRR and SLR requirements as other banks are.
  3. The source of their funds include deposits, central and state government, RBI, NABARD and other cooperative institutions.

Select the correct code:

A
1 and 2 Only
B
1 and 3 Only
C
2 and 3 Only
D
All of the above

112

Recently, ‘White Label ATM’ is in news; choose the correct statement/s with respect to White Label ATM:

  1. RBI is directly involved in White Label ATMs but is not the case with the Brown Label ATMs.
  2. White Label ATMS are not owned by the bank but are the properties of non-banking entities.

Choose the correct statement/ from the code given below:

A
1 Only
B
2 Only
C
Both 1 and 2
D
Neither 1 nor 2

113

Recently, UPI 2.0 (Unified Payment Interface) was launched. This is regulated by:

A
NPCI
B
SEBI
C
RBI and NPCI
D
RBI

114

Recently, “Bondi Bond” is in news; this is related to:

A
Bond issued by RBI and is named after a place in Rajasthan.
B
Bond issued by IMF that works on Blockchain Technology.
C
Bond issued by US government to deal with Trade war.
D
Bond issued by World Bank based on Blockchain Technology.

115

What does the term “Seigniorage” mean?

A
Difference between the value of new notes printed by RBI and the cost of printing and distribution.
B
Surplus with the RBI which it gives as Dividend to Government.
C
Adhering to BASEL-III norms.
D
Maintaining Capital Adequacy Ratio of 9%.

116

Consider the following statements about Prompt Corrective Action (PCA) which is in news:

  1. PCA framework is applicable to all Commercial Banks and Cooperative Banks.
  2. The PCA framework specifies the trigger points or the level in which the RBI will intervene with corrective action.
  3. Under PCA, RBI can change the Bank Board too.

Choose the correct code

A
2 Only
B
3 Only
C
1, 2 and 3
D
2 and 3 only

117

Consider the following statements about “Prevention of Money Laundering Act (PMLA):

  1. The Enforcement Directorate (ED) has power to claim possession only after the verdict of “Appellate Tribunal”.
  2. The accused can challenge the order of Adjudicating Authority to the “Appellate Tribunal” only after 180 days.

Choose the correct code

A
Only 1
B
Only 2
C
Both 1 & 2
D
Neither 1 nor 2

118

Which of the following is the tool of RBI for the Liquidity Adjustment Facility (LAF)?

  1. CRR
  2. SLR
  3. Repo Rate
  4. Reverse Repo

Choose the correct code :

A
1 and 2 Only
B
2 and 3 Only
C
1, 2, 3 and 4
D
3 and 4 Only

119

Which of the following is correct about “Soft Currency”?

A
Currency value is relatively constant compared to other currencies.
B
Digital Currency is sometimes referred to as Soft Currency.
C
Currency value fluctuates, predominantly lower, as a result of the country's political or economic uncertainty?
D
Currency based upon “Blockchain Techology” is called as Soft currency.

120

Cash reserve ratio (CRR) is commonly used monetary instrument to affect monetary policy. Increase in CRR leads to

  1. Reduction in the number of loans being made by banks.
  2. Reduction in liquidity.
  3. Reduction in interest rates.
  4. Reduction in consumer spending.

Select the correct code:

A
1, 2 and 3 Only
B
1, 2 and 4 Only
C
1, 3 and 4 Only
D
2, 3 and 4 Only

121

Which of the following committees was assigned to recommend reforms in the insurance sector?

A
Rekhi Committee
B
Nadkarni Committee
C
Malhotra Committee
D
Chelliah Committee

122

Which of the following institutions are fully owned subsidiaries of RBI?

  1. National Housing Bank.
  2. NABARD.
  3. Deposit Insurance & Credit Guarantee Corporation of India.
  4. SIDBI.
A
I, II and IV
B
Only I, II and III
C
Only III
D
All of the above

123

Choose the correct statements regarding NABARD:

  1. It acts as the regulatory authority for cooperative banks and regional rural banks
  2. NABARD operates the Rural Infrastructure Development Fund (RIFD)
  3. It is the National Implementing Entity (NIE) of Adaptation Fund under UNFCCC

Select the correct code:

A
1 only
B
1 and 2
C
2 only
D
1, 2 and 3

124

Which of the following is/are true for microfinance institutions?

  1. The first example of an organized microcredit institution is generally accepted as being the Grameen Bank in Bangladesh in 1976.
  2. It refers to an array of financial services, including loans, savings and insurance, available to poor entrepreneurs and small business owners who have no collateral and wouldn't otherwise qualify for a standard bank loan.

Select the correct code:

A
1 only
B
2 only
C
Both 1 and 2
D
None

125

Who releases WPI?

A
Office of the Economic advisor
B
CSO
C
NSSO
D
Ministry of Statistics and programme implementation

126

Which of the following is not a first generation reform in the economic front?

A
Reduction in statutory ceiling limits by RBI.
B
Disinvestment of Public Sector Enterprises.
C
Labour law flexibility.
D
Deregulation of Rupee.

127

Consider the following statements:

  1. Wholesale Price Index excludes both goods and services.
  2. Consumer Price Index includes both goods and services.

Which of the above statements regarding the indicators of price rise is/are correct?

Select the correct code

A
Both correct
B
2 Only
C
1 Only
D
None

128

Consider the following:

  1. GDP growth rate
  2. Foreign investment
  3. Political stability
  4. Literacy rate

Which of these can affect the value of sensex?

A
1 and 2
B
1, 2 and 3
C
All of the above
D
None of the above

129
Ways and Means Advances (WMA) system is meant for:
A
Temporary credit from RBI to the governments
B
Long-term credit from RBI to the government
C
All types of credit from RBI to the government
D
None of the above

130

Consider the following statements:

  1. The repo rate is the rate at which the RBI borrows money from commercial banks.
  2. Reverse Repo rate is the rate at which other banks borrow from the Reserve Bank of India.
  3. In India, RBI uses repo and reverse repo techniques to increase or decrease the liquidity in the market.
  4. Repo and reverse rates techniques can also be used to manage liquidity if there is an excessive capital flow from abroad.

Which of the statements given above is/are incorrect?

A
1 and 2
B
3 only
C
1, 2 and 4
D
1, 2 and 3

131

What is the correct chronological sequence of the establishment of the following institutions in India?

  1. NABARD
  2. IDBI
  3. SIDBI
  4. IFC1

Codes:

A
4, 1, 2, 3
B
3, 2, 1, 4
C
3, 1, 2, 4
D
4, 2, 1, 3

132

The monetary authority, typically the central bank of a country, is vested with the responsibility of conducting monetary policy. A monetary policy:

  1. Refers to the use of fiscal instruments of revenue and expenditure, to regulate the availability, cost and use of money and credit.
  2. The primary goal of monetary policy is growth, while keeping in mind the objective of price stability.

Choose the correct option:

A
Only 1
B
Only 2
C
Both 1 and 2
D
Neither 1 nor 2

133

Consider the following statements regarding the Non-performing assets:

  1. An asset, becomes non-performing when it ceases to generate income for the bank.
  2. A doubtful asset was one, which remained NPA for a period exceeding two years.
  3. Non-performing Asset (NPA) shall be an advance where interest and/or installment of principal remain overdue for a period of more than 180 days in respect of a Term Loan.

Choose the correct option from the codes given below:

A
I and II
B
II and III
C
I and III
D
all

134

As per the guidelines of RBI, which of the following are Non Performing Assets (NPA) or Bad loans?

  1. The instalment of principal or interest thereon remains overdue for two crop seasons for short duration crops.
  2. The instalment of principal or interest thereon remains overdue for one crop season for long duration crops.
  3. Interest and/or instalment of principal remains overdue for a period of more than 180 days in respect of a term loan.

Codes:

A
I and II
B
I and III
C
II and III
D
All of the above.

135

Which of the following functions are performed by Reserve Bank of India?

  1. To regulate or control the financial institutions.
  2. Issue of Currency Notes
  3. Determining subsidies
  4. Rupee sterilization

Select the correct answer using the code given below.

A
I and II only
B
I, II and IV
C
III and IV
D
All

136

Which of the following is not a feature of Gilt Edged Securities?

A
Assured returns on investment.
B
Higher rate of returns.
C
Company unlikely to default on payments.
D
None of the Above

137

Consider the following statements.

  1. All Micro Finance Institutions comes under the regulation of MUDRA Bank.
  2. Government has approved conversion of MUDRA Ltd. in to MUDRA-SIDBI Bank.
  3. Mudra scheme provides refinance for three types of loans i.e. Shishu Loan, Kishor Loan and Trun Loan.

Which of the statement given above is/ are correct?

A
I and III
B
II and III
C
II only
D
III only

138

Which of the following has been included under Priority Sector recently by the RBI?

A
Medium Manufacturing Enterprises.
B
Infrastructure sector
C
Retail trade
D
Petroleum sector

139

Which of the following is the primary deficit?

A
Fiscal Deficit – Payment of Interest
B
Budget Deficit – Payment of Interest
C
Capital Losses – Payment of Interest
D
Revenue Deficit – Payment of Interest

140

Consider the following:

  1. National Securities Depository Limited
  2. National Informatics Centre

Which of these facilitate demat trading in the stock exchanges?

A
Only 1
B
Only 2
C
Both 1 and 2
D
Neither 1 nor 2

141

Which of the following is called Budget Deficit?

A
Expenditure exceeding revenue receipt
B
Total expenditure exceeding total receipts
C
Both
D
None of the above

142

Consider the following.

  1. Regulation of the securities markets.
  2. Development of the securities markets.
  3. Protection of the interest of the investors.

Which of these are the functions of SEBI?

A
Only 1
B
1 and 3
C
1 and 2
D
All of the above

143

Which of the following statements is true related to Fiscal Deficit?

A
Difference between the Government’s total receipts and total fiscal expenditure
B
Difference between the Government’s total expenditure and total revenue excluding money from borrowings
C
Difference between the Government’s total revenue expenditure and borrowings
D
None of the above

144

Gross fiscal deficit includes:

A
Net domestic borrowings
B
Borrowings from the Government
C
External borrowings
D
All of the above

145

Which of the following is correctly matched?

A
Bombay Stock Exchange :NIFTY
B
National Stock Exchange :NYSE
C
Korean Stock Exchange: KOSPI
D
London Stock Exchange: NIKKEI

146

The index of a stock exchange measures the change in:

A
Size and performance of the selected companies.
B
Share prices of all the companies listed in the stock exchange.
C
Value of real assets of the selected companies.
D
Market capitalization of the selected companies.

147

Consider the following statements:

  1. Fiscal deficit is a crucial variable for the sustainability of the economy and the strong financial system of the public sector
  2. Revenue deficit is not a part of fiscal deficit
  3. Large part of the fiscal deficit shows that a large part of the loan is used for consumption expenditure instead of investment

Which of the above statements is/are true?

A
Only 1
B
1, 2
C
1, 3
D
All of the above

148

What does the primary deficit indicate?

A
Current fiscal imbalance
B
We get this by subtracting interest payment from the fiscal deficit
C
This indicates the Government’s net borrowing liabilities
D
Both (a) and (b)

149

Which of the following is/are primary equity market instrument(s)?

  1. Initial Public Offer.
  2. Further Public Offer.
  3. Futures.
  4. Exchange Traded Funds.

Select the correct answer using the codes given below:

A
Only I
B
I and II
C
I and III
D
I, II and IV

150

What can be the probable impact of ‘twin deficit’ in case of Indian economy?

  1. Crowding out of Private Investment
  2. Devaluation of Rupee
  3. High Inflation.

Choose the correct code(s)

A
II and III
B
I and II
C
I, II and III
D
II only

151

What are sovereign bonds?

A
Sovereign bonds are bonds held by sovereign countries which can be traded as foreign exchange.
B
Sovereign bonds are bonds floated by sovereign countries which can be traded as foreign exchange.
C
Sovereign bonds are bonds floated by some strong industrialized countries which are bought by other countries and are considered akin to foreign exchange reserves.
D
Sovereign bonds are bonds floated by sovereign countries in any currency which can be freely traded in the open market.

152

Non convertible debentures are:

A
Debt instruments which acquire equity status only at the investors’ option.
B
Debt instruments which acquire equity status only with the permission of Registrar of Companies.
C
Debt instruments which acquire equity status only on maturity.
D
Debt instruments which retain their debt character and cannot acquire equity status.

153

NRE deposit is:

A
Non Resident External deposit
B
Non Resident Exchange deposit
C
Non Refundable External deposit
D
Non Resident Extended deposit

154

What is Primary Deficit in the context of Government's finances?

A
It is the difference between what the government earns and its total expenditure (excluding the interest payments).
B
It is the difference between all non debt creating capital receipts and the total expenditure.
C
It is the difference between revenue receipts and revenue expenditure.
D
It is the difference between revenue receipts and revenue expenditure (excluding interest payments).

155

Consider the following statement regarding Payment Banks.

  1. Deposit and withdrawal facilities.
  2. Remittances.
  3. Providing loans.

Which of the above facilities can be provided by payments banks?

A
I only
B
I and II only
C
I and III only
D
I, II and III

156

Consider the following statements about Real time Gross settlement (RTGS) system.

  1. It operates on an order by order basis settlement system.
  2. It is primarily meant for large value transactions.
  3. RTGS system is also known as National Electronic Fund Transfer (NEFT) system.

Which of the statement(s) given above is/are TRUE?

A
I only
B
I and II
C
I and III
D
All of the above

157

What is meant by ‘high street banking’ in banking parlance?

A
Banks providing door-to-door services.
B
Home banking facility provided to privileged customer.
C
Emphasis by banks on retail lending.
D
Loan given to priority sector.

158

Which of the following policies will help in reducing a budget deficit?

  1. Increase government spending
  2. Reduce tax rates
  3. Promote economic growth
  4. Monetary easing

Choose the correct option:

A
1 and 2
B
1, 2 and 3
C
3 only
D
All of the above

159

Consider the following statements in the context of Bank rate:

  1. It is official rate of interest charged by the Reserved Bank of India on loans to other banks.
  2. It is the rate at which RBI discounts first class securities, including bills of exchange.
  3. It is also known as discount rate.
Which of the above statements is/are correct?
A
Only 1
B
Only 2
C
2 and 3
D
1, 2 and 3

160

Consider the following statements about the history of banks in India:

  1. Reserve Bank of India was set up on the basis of the recommendations of the Hilton Young Commission in 1935 and finally nationalized in 1949.
  2. Bank of India, founded in 1906 in Mumbai was the first Indian bank to open a branch outside India in London in 1946 and the first to open a branch in continental Europe at Paris in 1974.
  3. Canara Bank is the first bank in India to be given an ISO Certification.

Which of the above statements is/are false?

A
Only 1
B
Only 2
C
Only 3
D
None

161

What are the risks posed by fiscal deficit?

  1. It may cause macroeconomic instability by inflating the economy.
  2. Excessive capital for private sector.
  3. Higher taxes may hurt inter-generational parity.

Choose the correct option:

A
1 and 3
B
Only 1
C
Only 2
D
All of the Above

162

The Bombay Stock Exchange (BSE) has launched BSE Carbonex, the first carbon-based thematic index in the country, which takes a strategic view of organizational commitment to climate change mitigation. Which of the following statements are true?

  1. It will enable investors to track performance of the constituent companies of BSE-50 index regarding their commitment to greenhouse gases emission reduction.
  2. The American High Commission in India assisted in the development phase of the index.
  3. This index has been launched with the aim of creating a benchmark, and increasing awareness about the risks posed by climate change.

Codes:

A
Only 1
B
Only 2
C
Only 3
D
All of the above

163

Which of the following are targeted by the government to achieve fiscal sustainability?

  1. Primary deficit
  2. Debt to GDP ratio
  3. Revenue deficit
  4. Economic growth

Code:

A
1 and 2
B
1 and 3
C
1, 2 and 3
D
All of the above

164

Which of the following statements clearly defines the difference between preference shares and equity shares?

  1. The rate of dividend on equity shares may vary from year to year depending upon the availability of profit. Preference share- holders are paid dividend at a fixed rate.
  2. Equity shareholders do not enjoy voting rights. Preference shareholders have the right to participate in the management of the company.
  3. Dividend to equity shareholders is payable only after the preference shareholders have been paid.

Codes:

A
Only 2
B
1 and 3
C
1 and 2
D
Only 1

165

Which of the following is NOT a quantitative credit control measure of the Reserve Bank of India?

  1. Marginal Requirement of Loan
  2. Statutory Cash Reserve Requirements
  3. Statutory Liquidity Ratio
  4. Rationing of Credit

Codes:

A
2 and 3
B
4 only
C
1 and 4
D
None of the above

166

Which of the following is not a liability of banks?

A
Share capital
B
Reserve funds
C
Cash in hand
D
Deposits

167

Identify the drivers of government debt.

  1. The exchange rate movements.
  2. The level of the primary deficit.
  3. The difference between the interest rate on government debt and the growth of nominal GDP.

Code:

A
1 and 2
B
2 and 3
C
All of the above
D
1 and 3

168

Which of the following is not true about bank cheques?

A
The bearer cheque is payable to the person specified therein or to any other person who presents it to the bank for payment.
B
An order cheque is payable to the person specified therein as the payee, or to any one else to whom it is endorsed (transferred).
C
A crossed cheque cannot be encashed at the cash counter of a bank but it can only be credited to the payee’s account.
D
If a cheque bears a date earlier than the date on which it is presented to the bank, it is called as “ante-dated cheque”. Such a cheque is valid up to three months from the date of the cheque.

169

Building fiscal capacity is important for India's growth, but this will also require creating legitimacy in the state. Which of the following factors will aid in the process?

  1. The government’s spending priorities must include essential services that all citizens consume.
  2. Reducing corruption
  3. Scaling back subsidies to the well-off
  4. Property taxation needs to be developed.

Code:

A
1 and 2
B
2 only
C
1, 2 and 3
D
All of the above

170

Which of the following is not included in priority sector lending?

A
Housing loan
B
Micro credit
C
Education loan
D
Export Credit by domestic commercial banks

171

Which of the following is/are the implication of the fiscal deficit?

  1. National debt for future generations
  2. Inflation trap
  3. Erosion of Government’s credibility

Codes:

A
Only 1
B
Only 2
C
1 and 2
D
All of the above

172

What is the core objective of ‘Sterilization’ conducted by Reserve Bank of India?

A
To bring out the black money into the open
B
Prevention of depreciation/appreciation of domestic currency
C
Decreasing Liquidity in Economy
D
Reducing Current account deficit.

173

The difference between fiscal deficit and interest payment is called

A
Revenue Deficit
B
Primary Deficit
C
Budgetary Deficit
D
Capital Deficit

174

Which of the following is /are tool(s) of Reserve Bank of India for qualitative credit control?

  1. Credit Rationing
  2. Liquidity Adjustment Facility
  3. Variation in Bank Rate.

Which of the statement(s) given above is/are correct?

A
II and III
B
I Only
C
I and II
D
I, II and III

175

Which of the following is Not True about the Reserve Bank of India?

A
It regulates the currency and credit system of India.
B
It maintains the exchange value of the rupee.
C
Foreign exchange reserves are kept by RBI.
D
Coins are issued by RBI.

176

The fiscal deficit means

A
Total Expenditure – Gross receipts excluding lending
B
Revenue Expenditure – Revenue Receipts
C
Capital Expenditure – Capital Receipts
D
Revenue Expenditure + Capital Expenditure – Revenue Receipts

177

Which of the following is/are not a qualitative measure of credit control?

  1. Rationing of credit.
  2. Variation of Reserve ratio.
  3. Regulation of consumer credit.
  4. Variation of Margin requirements.

Select the correct answer using the codes given below:

A
II, III and IV
B
II and IV
C
II and III
D
II Only

178

Which of the following is correct about ‘Merchant Banks’?

A
They are owned by big corporate houses.
B
They deal only with corporates.
C
Loan is given on a lower rate of interest.
D
They offer various services other than banking.

179

Consider the following statements-

  1. Banking sector in India is regulated by the reserve Bank of India.
  2. Ministry of finance has authority to give license to new bank. Then RBI ratifies that bank.

Which of the statement given above is/are incorrect?

A
I Only
B
II Only
C
Both
D
None

180

What does a fiscal deficit indicate?

A
It indicates how much government borrowing is going to meet expenses excluding the interest payments
B
It reflects the extent to which current government policy is adding to future burdens stemming from past policy.
C
It reflects how far the exchequer is living beyond its means.
D
It highlights the information on what the government is borrowing for

181

What fiscal measures can help in decreasing the aggregate demand?

  1. Reducing government expenditure.
  2. Decreasing the amount of taxes.
A
1 only
B
2 only
C
Both
D
None

182

What is fiscal deficit?

A
Total Expenditure - Total Receipts excluding borrowings
B
Revenue Expenditure - Revenue Receipts
C
Receipts and expenses in revenue account - Receipts and expenses in capital account
D
Revenue Deficit - Interest Payments

183

Primary deficit is defined as

A
Fiscal deficit adjusted for interest payments
B
Revenue deficit adjusted for loans
C
Fiscal deficit adjusted for loans
D
Revenue deficit adjusted for interest payments

184

Deficit financing is a method of meeting government deficits. Through which of the following ways is the deficit financing done in India?

  1. Withdrawal of past accumulated cash balances by the government
  2. Borrowing from RBI
  3. Printing fresh currency notes
  4. External Commercial Borrowing

Select the correct code:

A
1, 2 and 3
B
1, 2 and 4
C
2, 3 and 4
D
1, 2, 3 and 4

185

Which of the following measures help in achieving fiscal consolidation?

  1. Increasing taxes and ensuring tax compliance.
  2. Borrowing from the market by the government.
  3. Reducing subsidies.

Select the correct code:

A
1 and 2
B
1 and 3
C
2 and 3
D
1, 2 and 3

186

Consider the following statements is/are correct?

  1. Primary deficit is the difference between fiscal deficit and interest payments.
  2. Monetized deficit is the borrowings made from RBI through printing fresh currency.

Select the correct code:

A
1 only
B
2 only
C
Both 1 and 2
D
Neither 1 nor 2

187

Which among the following is/are part of ‘deficit financing’ as used in the economic planning of India:

  1. Borrowing from Commercial Banks
  2. Disinvestment in Public Sector Company
  3. Issuing fresh currency notes
  4. Bailing out the Banks.

Choose the correct code:

A
2, 3 and 4 Only
B
1, 2 and 3 Only
C
3 Only
D
3 and 4 Only

188

Which of the following, if adopted in policy making, would promote ‘fiscal consolidation’:

  1. Increasing FDI in Insurance
  2. Enactment of food security laws
  3. Direct Benefit Transfer.

Choose the correct code(s)

A
2 and 3 Only
B
1 and 2 Only
C
2 Only
D
1 & 3 Only

189

What is Net primary deficit?

A
It is defined as net fiscal deficit minus net interest payments.
B
It is gross primary deficit minus net tax payment.
C
It is defined as revenue deficit minus that revenue expenditure (in the form of grants), which goes into the creation of Capital Assets.
D
It is that part of the government’s deficit which is financed through short-term borrowings.

190

Due to recent rise in crude oil price, it is said that it will lead to “Twin Deficit”. Twin Deficit means deficit on account of:

A
Capital Account Deficit and Fiscal Deficit (FD)
B
Current Account Deficit (CAD) and FD
C
CAD and Revenue Deficit
D
CAD and Effective Revenue Deficit

191

Consider the following statements about “Fiscal Deficit”:

  1. It is the borrowing requirement of the Government.
  2. Fiscal Deficit is always a negative indication for a country’s economy.
  3. The Union Budget 2018 has pegged Fiscal Deficit at 3.3%.

Choose the correct statement/s from the codes given below:

A
2 only
B
1, 2 and 3
C
1 and 2 Only
D
1 and 3 Only

192

Consider the following statements-

  1. Continuous declining in price of crude oil will boost the economic growth of India.
  2. It will also help to reduce the fiscal deficit of the country.

Which of the statement given above is/are correct?

A
I Only
B
II Only
C
Both
D
None

193

Which of the following statements regarding the importance of Disinvestment in India is/are correct?

  1. It helps in financing the increasing fiscal deficit.
  2. It helps in financing large scale infrastructure development.
  3. It is important for investing in the economy to encourage spending.
  4. It is important for reducing Government debt.
  5. It is important for implementing social programmes like health and education.

Codes:

A
I, II, III and IV
B
I, II, IV and V
C
II, III and V
D
All of the above.

194

Which one of the following budgetary deficit is highly inflationary?

A
Fiscal deficit
B
Revenue deficit
C
Primary deficit
D
Monetized deficit

195

Consider the following statements:

  1. RRB’s were established to take the banking services to the doorsteps of rural masses, especially in remote rural areas with no access to banking services.
  2. These banks were intended to mobilize rural savings and channelize for supporting the productive activities in the rural areas.

Which of the above statements is/are correct?

A
Only I
B
Only II
C
Both
D
None

196

Which of the statements about Repo rate is/are correct?

  1. It is the sale of securities together with an agreement for the seller to buy back the securities at a later date.
  2. The repurchase price should be greater than the original sale price and the difference is repo rate.

Codes:

A
Only I
B
Only II
C
Both
D
None

197

Which of the following statements is/are correct?

  1. When the private or public banks are in need of money, they borrow from RBI at bank rate.
  2. If the bank rate increases money circulation decreases.

Codes:

A
Only I
B
Only II
C
Both
D
None

198

Match List 1 with List 2 and select the correct answer using the codes given below the Lists.

A. Fiscal deficit 1. Excess of total Expenditure over Total Receipts
B. Budget deficit 2. Excess of Revenues Expenditure over revenue Receipts
C. Revenue deficit 3. Excess of Total Expenditure over total Receipts less borrowings
D. Primary deficit 4. Excess of Total Expenditure over total receipts less borrowings and interest payment.

Codes:

A B C D

A
3 1 2 4
B
3 1 4 2
C
1 3 2 4
D
1 3 4 2

199

Consider the following statements:

  1. Cash Management Bills (CMBs) are short-term papers with the flexibility of fixing tenure according to the requirement of the government.
  2. CMBs can have tenure between seven days and one year.

Which of the above statement is/are correct?

A
I only
B
II only
C
Both
D
None

200

Which of the following correctly explains ‘effective revenue deficit’ introduced by the government of India in the Union Budget of 2011-12?

A
Revenue Deficit - Borrowings
B
Revenue Deficit – grants for creation of capital assets
C
Revenue Deficit – Interest Payments
D
Revenue Expenditure- Fiscal Deficit.

201

Consider the following statements about the Non performing Assets (NPA):

  1. Any asset, including a leased asset, becomes a non performing when it ceases to generate income for the bank and are called Non Performing Assets.
  2. As per RBI guidelines, NPA does not include short duration crops loan.

Which of the above statements is/are correct?

A
I only
B
II only
C
Both
D
None

202

Effective revenue deficit is a new concept recently introduced in the budgetary process of India. Which of the following statements about effective revenue deficit are correct?

  1. Effective revenue deficit is more than revenue deficit.
  2. Effective revenue deficit is less than revenue deficit.
  3. Effective revenue deficit essentially tells us about purely consumption component of revenue deficit.

Codes:

A
Only 1
B
Only 2
C
Only 2 and 3
D
Only 3

203

Which of the following are Public Key Infrastructure enabled electronic payment systems that have been introduced by the RBI?

  1. Government Securities Clearing.
  2. National Electronic Funds Transfer (NEFT).
  3. Real time gross settlement systems (RTGS).

Codes:

A
I and II only
B
II and III only
C
I and III only
D
All

204

Which of the following are financial risks to which the systems of payment are subjected to?

  1. Credit risk
  2. Operational risk
  3. Liquidity risk

Codes:

A
I and II only
B
II and III only
C
I and III only
D
All

205

Deficit financing, sometimes referred to as a budget deficit, is an approach to money management that involves spending more money than is collected during the same period. Why do most developing countries, including India, resort to deficit financing?

  1. To increase aggregate demand through increased public expenditure.
  2. To mobilize surplus, ideal and unutilized resources in the country.
  3. To finance defense expenditure.
  4. To contain inflation.

Select the correct answer from the codes given below:

A
1 only
B
1 and 2
C
1, 2 and 3
D
All the above

206

Consider the following statements about Nachiket Mor Committee Report:

  1. It has recommended that Reserve Bank of India (RBI) should gradually increase the Statutory Liquidity Ratio (SLR).
  2. Interest subsidies and loan waivers should be encouraged.

Which of the above statements is/are correct?

A
I only
B
II only
C
Both
D
None

207

Which among the following is/are true?

  1. Deficit financing does not lead to inflation if adopted in small doses.
  2. Deficit financing is an often used tool for financing budgetary deficits.

Which of the statements given above is/are correct?

A
Only I
B
Only II
C
Both I and II
D
None of the above

208

What is Primary Deficit in the context of Government’s finances?

A
It is the difference between what the government earns and its total expenditure (excluding the interest payments).
B
It is the difference between all non debt creating capital receipts and the total expenditure
C
It is the difference between revenue receipts and revenue expenditure.
D
It is the difference between revenue receipts and revenue expenditure (excluding interest payments).

209

Which of the following are not responsible for the fiscal deficit enhancement?

  1. Interest payment
  2. Subsidies
  3. Defence expenditure

Choose the correct options:

A
I and II
B
II and III
C
I, II and III
D
none of these

210

Under which of the following agreements between the Government of India and RBI, monetization of the deficit was curtailed?

A
Ways and Means Agreement.
B
Financial Prudency and Austerity Agreement.
C
Fiscal Consolidation and Fiscal Discipline Agreement.
D
Monetary Means and Fiscal Ways Agreement.

211

If we subtract Revenue receipts and Non- debt creating capital receipt from the total expenditure, we get

A
Budget deficit
B
Revenue deficit
C
Fiscal deficit
D
Monetized deficit

212

Which one of the following is/are the possible effects of deficit financing?

  1. The rise in the level of incomes
  2. The fall in the general price level
  3. Increase in money supply with the public

Select the correct answer from the codes given below:

A
I, II and III
B
I and III only
C
I only
D
III only

213

Consider the following statements: A government may cover up the deficit by

  1. Withdrawing its cash balances from the central bank of the country.
  2. Borrowing from the central bank and commercial banks.
  3. Printing new money.

Of these statements

A
1 and 2 are correct
B
1, 2 and 3 are correct
C
1 and 3 are correct
D
2 and 3 are correct

214

Which among the following is/are part of ‘deficit financing’ as used in the economic planning of India

  1. Borrowing from Commercial Banks
  2. Disinvestment in Public Sector Company
  3. Issuing fresh currency notes
  4. Bailing out the Banks.

Choose the correct code(s)

A
II, III and IV
B
I, II and III
C
III Only
D
III and IV

215

Consider the following statements Regional Rural Banks in India:

  1. Are required to assist the weaker sections.
  2. Are to help the small and marginal farmers.
  3. Have limited areas of operation.
Which of the above statements are correct?
A
I and II
B
II and III
C
I and III
D
All

216

Which of the following statements regarding objectives for maintaining the SLR ratio is/are correct?

  1. To control the expansion of bank credit.
  2. To ensure the solvency of commercial banks.
  3. To compel the commercial banks to invest in government securities like government bonds.

Codes:

A
I and II
B
I and III
C
II and III
D
All of them

217

Which of these statement is/are not true about RBI:-

  1. It acts as bank of Central Government and all State Governments
  2. It is sole authority to issue currency
  3. It is Banker’s Bank
  4. It is Lender of Last Resort to banks

Code:

A
I and II
B
II and III
C
I and IV
D
none of the statements

218
Which of the following will be a direct action by RBI?
A
Increase in Bank rate
B
Refuse to refinance a Bank
C
Decrease in Repo rate
D
Purchase of foreign reserves

219

Consider the following statements about SLR (Statutory Liquidity Ratio):

  1. It is that ratio of the total deposits of a bank which it has to maintain with itself in the form of liquid funds like government securities and cash in hand.
  2. SLR is exclusive to India. No other country has SLR.
  3. It helps the government to borrow money from banking system.

Which of the above statements are correct?

A
I, II only
B
I, III only
C
II, III only
D
all

220

This phenomenon occurs when low or zero interest rates fail to stimulate consumer spending and monetary policy becomes ineffective.

A
Liquidity Trap
B
Quantitative Easing
C
Fed Tapering
D
Dutch disease

221

Which of the following constitute as an Instrument of Selective Credit control in India?

  1. Bank Rate
  2. CRR
  3. Open market operation.

Choose the correct options from the codes given below:

A
I and II
B
II and III
C
All
D
None of the above

222

As the supreme banking authority in the country, the Reserve Bank of India has the following powers:

  1. It holds the cash reserves of all the scheduled banks.
  2. It controls the credit operations of banks through quantitative and qualitative controls.
  3. It controls the banking system through the system of licensing, inspection and calling of information.
  4. It acts as the lender of the last resort by providing rediscount facilities to scheduled banks.

Which of the above stated powers are correct?

A
I, II, III only
B
II, III, IV only
C
I, III, IV only
D
All

223

Consider the following statements:

  1. Repo (Repurchase) rate is the rate at which banks park their short-term excess liquidity with the RBI.
  2. Reverse Repo rate is the rate at which the RBI lends shot-term money to the banks.

Which of the above statements is/are correct?

A
I only
B
II only
C
Both
D
None

224

The Financial Stability Board (FSB)

A
It serves central banks in their pursuit of monetary and financial stability, to foster international cooperation in those areas and to act as a bank for central banks.
B
It is an autonomous body dealing with macro prudential andfinancial regularities in the whole financial sector of India.
C
It fosters global monetary cooperation, secure financial stability, facilitate international trade, promote high employment and sustainable economic growth, and reduce poverty around the world.
D
It is an international body that monitors and makes recommendations about the global financial system.

225

Consider the following statements:

  1. Bank Rate implies the rate of interest at which the RBI discounts the Bills of exchange.
  2. Bank rate is the rate of interest charged by commercial banks on loans.

Select the correct statement from the codes given below:

A
Only I
B
Only II
C
both
D
None

226

Consider the following statements about Follow-on Public Offer (FPO):

  1. It is an issuing of shares to investors by a public company that is already listed on an exchange.
  2. Follow-on Public Offer is both dilutive and Non-dilutive.
  3. It allows the private companies only to raise additional equity capital through a stock issue.

Which of the above statements is/are correct?

A
I and II only
B
II and III only
C
I only
D
all

227

Consider the following statements about the NBFC- Non- banking financial companies:

  1. NBFC are engaged in the business of loans and advances, acquisition of shares/stocks/bonds/debentures/securities.
  2. NBFCs do not form part of the payment and settlement system but can issue cheques drawn on itself.
  3. Deposit insurance facility of Deposit Insurance and Credit Guarantee Corporation is not available to depositors of NBFCs.

Which of the above statements is/are correct?

A
I and II only
B
II and III only
C
I and III only
D
All

228

Which of the following are the benefits of RuPay platform?

  1. It helps in reducing the number of cash transactions.
  2. It reduces the transaction cost.
  3. It helps in reaching to the unbanked sections.
  4. It will be available at a cost much less than those of international cards.

Codes:

A
I and II only
B
I, II and IV only
C
II and III only
D
All are correct

229

The objectives of Statutory Liquidity Ratio are:

  1. To control the expansion of bank credit.
  2. To ensure the solvency of commercial banks.
  3. To compel the commercial banks to invest in government securities like government bonds.

Which of the aforesaid objectives are correct?

A
I and II
B
I and III
C
II and III
D
All of the above

230

Consider the following terms about quantitative credit control measure of the Reserve Bank of India:

  1. Statutory Liquidity Ratio.
  2. Cash Reserve Ratio.
  3. Rationing of Credit.

Which of the above stated measures are correct?

A
I, II only
B
II, III only
C
I, III only
D
all

231

Which of the following statements regarding Repo and Reverse Repo rate is/are correct?

  1. Repo and Reverse Repo are instruments used by RBI in day- to- day liquidity management under the Liquidity Adjustment Facility.
  2. A hike in Repo rate reduces credit and money supply in the economy and vice- versa for a fall in Repo rate.
  3. Repo rate is generally resorted to in order to fine tune the liquidity position in short run without resorting to major policy changes.

Codes:

A
I and II only
B
I and III only
C
II and III only
D
All of them

232

Which of the following are quantitative measures of credit control used by RBI?

  1. Bank Rate Policy
  2. Open Market Operation
  3. Consumer credit Regulation
  4. Cash Reserve Ratio

Select the correct answer using the code given below:

A
I and II
B
I, II and III
C
I, III and IV
D
I, II and IV

233

Non-Banking Financial Companies (NBFCs) are fast emerging as an important segment of Indian Financial System. Which of the following statement about NBFC is incorrect?

A
An NBFC can’t accept demand deposits.
B
NBFCs raise funds from the public directly or indirectly and lend them to the ultimate spenders.
C
Deposit insurance facility is not available for NBFC depositors unlike in case of Banks.
D
NBFCs can issue cheques drawn on itself.

234

Consider these statements regarding bank credit:

  1. Banks collect deposits from savers and lend these to investors and others.
  2. The deposits of banks form the basis of their lending operations.
  3. Banks use their deposits for advancing credit only but not for investment purpose.

The correct statement(s) is/are:

A
Only I
B
Only II
C
I and II
D
Only III

235

Which are the following means to stabilise the price rise, undertaken by government?

  1. Minimum support price
  2. Buffer stock
  3. State trading
  4. Decentralised Procurement Scheme

Select the answer from the codes given below:

A
I, II and III
B
II and III
C
I, III and IV
D
All of these

236

Consider the following statements regarding Micro Units Development Re-finance Agency (MUDRA) bank-

  1. It first began as a subsidiary of IDBI and later converted to a full fledged bank through an act of parliament.
  2. If banks fail to meet priority sector lending norms, they’ve to deposit money in NABARD’s RIDF or SIDBI’s SEDF.

Which of the above is/are correct?

A
1 only
B
2 only
C
Both 1 and 2
D
None of these

237

RBI and Finance ministry signed an agreement to implement recommendations of Urjit Patel Committee on monetary policy reforms. Which of the following points were agreed upon?

  1. Inflation targeting at CPI 2-6% i.e. @ 4% with band of 2%.
  2. Monetary policy to be decided by a Monetary Policy Committee.
  3. Among 3 functions of controlling- inflation, foreign exchange rate and interest rate, RBI will give top priority to inflation.
A
1 and 3
B
2 and 3
C
2 only
D
All of the above

238

Which of the following correct regarding Liquidity Adjustment Facility (LAF)-

  1. It was started in 2000 by RBI as a tool to control money supply in economy.
  2. Bankers cannot pledge securities from SLR quota to borrow from this window.
  3. Only scheduled commercial banks can borrow under this window.
A
1 and 2
B
2 and 3
C
3 only
D
All of the above

239

Which of the following are economic objectives of Fiscal Policy?

  1. To maintain and achieve full employment
  2. To stabilize the price level.
  3. To stabilize the growth rate of the economy.
  4. To maintain equilibrium in the balance of payments.

Select the answers from the codes given below:

A
1, 2 and 3 only
B
3 and 4 only
C
1, 3 and 4 only
D
All of the above

240

Consider the following statements:

  1. Base Rate is the rate above which banks cannot extend loans to either corporate or individual borrowers.
  2. Bank Rate is the rate of interest which a Central bank charges on the loans to a commercial banks for short term.
  3. Reverse Repo Rate is the rate at which the banks park surplus funds with Reserve Bank.
  4. Repo rate is the rate at which the banks borrow from other banks.

Which of the above statement are incorrect?

A
1 and 2
B
2, 3, and 4
C
1, 2, 3 and 4
D
2 and 4

241

Which of the following are the measures adopted by the central bank to control the purpose of the loans given by commercial bank?

  1. Priority sector lending
  2. Moral persuasion
  3. Publicity and direct action
  4. Open market operation

Select the correct options from the following codes:

A
1 only
B
2 only
C
1, 2 and 3
D
All of the above

242

The cost of bank credit is determined in India by a system of benchmark prime lending rate charged by banks. Of the following, what determined this rate of interest?

A
It is competitive rate determined by market forces of supply and demand for credit
B
It is fixed by the Reserve Bank of India
C
It is determined by the Bank concerned
D
It is determined on the basis of LIBOR rate.

243

Lead bank scheme is designed to

A
Control the activities of non banking financial institutions
B
Help the weaker sections in a particular district
C
Have more concentrated effort of a bank in a particular district
D
Regulate the use of credit

244

According to the RBI, which of the following mechanism is most effective to deal with rising Non-Performing Assets of Indian Banks?

A
Implementation of SARFESI Act
B
Asset Reconstruction Companies
C
Joint Lenders Forum
D
Strategic Debt Restructuring Scheme

245

The Reserve Bank of India released the Draft Guidelines on Net Stable Funding Ratio (NSFR) under Basel III Framework on Liquidity Standards for banks. Which of the following is/are correct about NSFR?

A
NSFR is amount of available stable funding relative to the amount of required stable funding.
B
It measures the amount of long-term, stable sources of funding employed by an institution relative to the liquidity profiles of the assets funded.
C
Both (a) and (b)
D
None of the above

246

Which of the following statement is incorrect with respect to recently launched MUDRA bank?

A
MUDRA Bank is a public sector Finan-cial institution in India.
B
It will refinance Micro Finance Institutions serving MSMEs.
C
The bank will initially function as a non-banking financial company .
D
MUDRA cannot extend refinance support to Regional Rural Banks for enhancing their liquidity.

247

Which of the following terms indicate a mechanism used by commercial banks for providing credit to the Government?

A
Cash credit ratio
B
Debit service Obligation
C
Statutory liquid ratio
D
Liquidity adjustment facility

248

Which one of the following statements about Payments Banks is incorrect?

A
Mobile phone companies can run a Payment bank.
B
Payment bank will have to keep CRR.
C
Payment bank cannot involve in any credit risk.
D
Payment bank can hold more than Rs.50,000 per customer.

249
Which of the following is CORRECT regarding Prime Lending Rate (PLR)
A
PLR is the rate at which bank lends to the Industries.
B
PLR is the rate at which banks lends to the Farmers.
C
PLR is the rate at which banks lends to the Government.
D
PLR is the rate at which banks lends to the Best Customers.

250

What are the advantages of Electronic Fund Transfer over the conventional one?

  1. Technology eliminates the risks associated with lost, stolen or misdirected cheques as it uses Bluetooth to transfer data.
  2. Allows for faster response time to payment inquiries.
  3. Mobile Money does not come under Electro-nic Fund Transfer technology.

Which of the above statements is/are correct?

A
Only II
B
I and III
C
II and III
D
All

251

Which of the following sectors are parts of Priority Sector Lending?

  1. Agriculture
  2. Small scale industries
  3. Education
  4. Low Cost Housing

Select the correct answer using the code given below.

A
I and II
B
I, II and III
C
I, III and IV
D
All of the above

252

Which of the following is not correct regarding separate platform that SEBI is planning to help startup companies get capital?

  1. Name of this platform is Alternate Capital Raising Platform.
  2. Investors that can participate here are Retail investors.
  3. Investors that can’t participate here are High net worth individuals, qualified institutional buyers, family trusts, NBFCs and other entities with 500 crore net worth.
A
1 and 3
B
3 only
C
2 only
D
2 and 3

253

Consider the following statement about Priority sector lending

  1. Priority sector guidelines of RBI lay down preferential rate of interest for priority sector loans.
  2. Failure to meet PSL target results in Banks investing in Rural Infrastructure Development Fund.
  3. PSL includes social infrastructure and renewable energy.

Which of the above statement(s) is/are correct?

A
Only 1
B
Only 2
C
1 and 2
D
2 and 3

254

Which of the following sector(s) form part of Priority Sector Lending?

  1. Agriculture
  2. Renewable Energy
  3. Social Infrastructure
  4. Medium Enterprises

Select the correct answer using the codes given below:

A
I only
B
I, II and III only
C
I and IV only
D
All of the above

255

Which of the following is/are correct regarding Priority Sector Lending (PSL)?

  1. Indian Banks have to lend 40 percent of their capital as PSL
  2. Foreign Banks are not covered under PSL norms
  3. Loans to minorities are part of PSL

Choose the correct code(s)

A
I Only
B
I and II
C
I and III
D
I, II and III

256
What is offshore banking?
A
It is a bank located in an island country e.g. Mauritius.
B
It is a bank located in SEZ.
C
It is a bank located outside the country of residence of the depositor.
D
It is a bank located in a neighbouring country.

257

Which of the following best defines a floating-rate bond?

A
A bond with a fixed interest rate and has better yield than varying interest rate bond.
B
A bond with a fixed interest rate and has lower yield than varying interest rate bond.
C
A bond with a varying interest rate and has better yield than fixed interest rate bond.
D
A bond with a varying interest rate and has lower yield than fixed interest rate bond.

258

Consider the following statements.

  1. On the behalf of RBI, Ministry of Finance, Department of Corporate Affairs can also Frame the Monetary policy and act upon it.
  2. Policy rate is greater than Bank rate and less than Repo rate.
  3. Reduction in the Policy rate will increases the flow of money in the market.

Which of the statement given above is /are correct?

A
I and III
B
II and III
C
II only
D
None

259

Which one of the following is not an instrument of the Indian money market?

A
Commercial bills
B
Debentures
C
Treasury bills
D
Certificate of Deposits

260

Who amongst the following benefits most from inflation?

A
Creditors
B
Debtors
C
Saving Bank Account Holders
D
Government Pensioners

261

A rise in the general level of prices may be caused by:

  1. An increase in the money supply
  2. A decrease in the aggregate level of output
  3. An increase in the wages and salaries. Of these statements
A
I alone is correct
B
I and II are correct
C
II and III are correct
D
I, II and III are correct

262

Which of the following are related to Objective of monetary policy?

  1. Accelerating growth of economy.
  2. Price stability.
  3. exchange rate stabilization.
  4. Balancing savings and Debits.
  5. Generating Employment.

Select the correct answer using the codes given below:

A
I, II, III and IV
B
I, III, IV and V
C
I, II, III and V
D
I, II, III, IV and V

263

The Cheap and Dear money policy relate to changes in

A
REPO rate
B
Bank rate
C
Both REPO rate and reverse REPO rate
D
Both REPO rate and Bank rate

264

Consider the following statements in the context of Money supply and monetary aggregates:

  1. Reserve Money (M0) = Currency in Circulation + Bankers’ deposits with the RBI + ‘Other’ deposits with the RBI.
  2. Narrow Money (M1) = Currency with the Public + Demand Deposits with the Banking System + ‘Other’ Deposits with the RBI.
  3. Broad Money (M3) = M1 + Time Deposits with the Banking System
  4. M4 = M3 + All deposits with Post Office Savings Banks (excluding National Savings Certificates).

Which of the statements given above are correct?

A
1, 2, 3 and 4
B
1, 2 and 4
C
2, 3 and 4
D
1, 3 and 4

265

Which of the following will result into increase of money supply in the economy?

  1. Sale of government securities to the public by RBI.
  2. Borrowing by the government from RBI.

Codes:

A
I Only
B
II Only
C
Both I and II
D
Neither I nor II

266

Which of the following statements is/are not correct about Follow-on Public Offer (FPO)?

A
It is an issue of shares to investors by a public company that is already listed on an exchange.
B
It is not essentially a stock issue of supplementary shares made by a company.
C
It is issued by a company which has gone through the Initial Public Offer (IPO) process.
D
It can be of two types Dilutive and Non-dilutive.

267

Which of the following will not signify Expansionary Monetary Policy?

  1. Reduction in Bank rate
  2. Reduction in CRR
  3. Increase in SLR
  4. Increase in Tax rate

Code:

A
I and IV
B
III only
C
IV only
D
none of them signify

268

Which of the following is the result of Dear Monetary Policy?

A
Increase in money supply
B
Inflation
C
Stagflation
D
None of above

269

Consider the following statements:

  1. Speculative demand for money is inversely related to the rate of interest.
  2. If the supply of money in the economy increases the rate of interest will also increase.
  3. The total demand for money in an economy is composed of transaction demand and speculative demand.
  4. Transaction demand is directly proportional to the real GDP and Price level.

Choose the incorrect statements from the codes given below.

A
Both I and II
B
Only II
C
Both III and IV
D
All are incorrect.

270

Consider the following statements regarding the narrow money:

  1. Currency (Notes plus Coins), demand deposit and saving deposits with post office savings banks constitute Narrow Money.
  2. Net time deposits with commercial banks is called narrow money.

Select the correct statement from the codes given below:

Codes:

A
Both I and II
B
Only I
C
Only II
D
Neither

271

Consider the following statements about Devaluation of Currency:

  1. ‘Devaluation’ means the official lowering of the value of a country’s currency within a fixed exchange rate system, with respect to major internationally traded currencies.
  2. Depreciation is used to describe a decrease in a currency’s value relative to other major currency benchmarks due to market forces, not government or central bank policy actions.
  3. Under the system of Depreciation central banks maintain the rates up or down only by buying or selling USD.

Which of the above statements are correct?

A
I and II
B
I and III
C
II and III
D
All of the above

272

Which of the following statement about Quantitative easing is incorrect?

A
Quantitative easing (QE) is a type of monetary policy used by central banks to stimulate the economy when standard monetary policy has become ineffective.
B
It is aimed to keep interbank interest rates at a specified target value.
C
It involves buying assets of longer maturity than short-term government bonds, thereby lowering longer-term interest rates.
D
All are correct.

273

Which of the following are measures of accommodative monetary policy?

  1. Decrease in Repo Rate.
  2. Increase in Cash Reserve Ratio.
  3. Increase in Statutory Liquidity Ratio.

Select the correct answer using the code given below.

A
I and II
B
I only
C
II and III
D
All of the above

274

Liquidity trap is a situation in which

A
Interest rates are low and savings rates are high.
B
Consumers choose to buy bonds.
C
Fluctuations in the money supply translates into fluctuations in price levels.
D
None of the above.

275

With reference of Monetary Policy Framework agreement announced between RBI and Union Government, which of the given statement(s) is/are correct?

  1. There is only one proposal to establish the Monetary policy Committee.
  2. The RBI will aim to bring inflation 6% by January 2016.
  3. Objective of monetary policy is to primarily maintain price stability while keeping in mind the objective of growth.

Which of the above statements are correct?

A
1 and 2
B
1 and 3
C
1, 2 and 3
D
2 and 3

276

What does it mean by the High Powered Money?

A
The total liability of the monetary authority of the country, RBI
B
The total liability of the commercial banks to the monetary authority
C
Term deposit of banks
D
None of the above

277

Which of the following is/are the instrument/s of the Monetary Policy?

A
Open Market Activities
B
Bank Rate policy
C
Cash Reserve Ratio and Statutory Liquidity Ratio
D
All of the above

278

Which of the following is the force of the inbound market policy?

  1. Import substitution
  2. Protecting domestic industries with foreign competition
  3. Export promotion

Choose the correct answer using the codes given below:

A
Only 1
B
Only 2
C
1 and 2
D
All of the above

279

Which of the following is/are correct regarding monetary aggregates?

A
M3 = M1 + Time deposits with commercial banks.
B
M1 is also known as broad money, while M3 is known as narrow money.
C
Ratio of M1 to M3 is known as Money multiplier.
D
All of the Above.

280

Which of the following constitutes "M3" or broad money?

  1. Currency with public
  2. Demand deposit in all banks
  3. Time deposits with commercial banks
  4. Post office bank savings
  5. Other deposits with RBI
A
1 & 2 only
B
1 & 4 only
C
1, 2, 3 & 5 only
D
All of the above

281

Depreciation is

  1. A real expenditure which occurs every year on investments.
  2. A virtual expenditure which occurs every year on investments.
  3. An accounting concept.
  4. A wear and tear loss which is accounted on investments.

Select the correct code:

A
1 only
B
1 and 3 only
C
2, 3 and 4 only
D
1 and 4 only

282

Which is the least liquid of the below?

  1. M1
  2. M2
  3. M3
  4. M4

Select the correct code:

A
1 only
B
2 only
C
4 only
D
3 only

283

Which of the following is correct?

  1. M1 = Currency held by public + net demand deposits held by commercial banks.
  2. M2 = M1 + saving deposits with Post Office savings banks.
  3. M3 = M1 + Net time deposits of commercial banks.
  4. M4 = M3 + Total deposits with Post Office savings organizations (excluding National Savings Certificates).

Select the correct code:

A
1 and 2
B
1 and 3
C
2 and 4
D
All of the above

284

Which of the following are instruments of monetary policy?

  1. Taxation
  2. Marginal Standing Facility
  3. Budget allocations
  4. Market Stabilisation Scheme

Select the correct code:

A
1, 2 and 3
B
2 and 4
C
2, 3 and 4
D
3 and 4

285

What do you mean by velocity of circulation of money?

A
The speed with which money is being issued by RBI.
B
The number of times a unit of money changes hands during the unit period.
C
The rate at which a customer withdraws his money from the bank.
D
The way in which the money travels between different financial institutions and intermediaries.

286

Consider the following statement with respect to Monetary Policy Committee (MPC):

  1. It is a six member body that has to meet at least 4 times a year and is chaired by RBI Governor.
  2. Rate is decided by voting and only in case of ‘tie’ the chairman will have the second vote or casting vote.

Choose the correct code:

A
Only 1
B
Only 2
C
Both 1 & 2
D
Neither 1 nor 2

287

What is the effect of appreciation of Indian Rupee on the Indian economy?

  1. Lower inflation
  2. Lowering of foreign debt
  3. Exports become competitive
  4. Reduction in trade deficit

Select the correct code:

A
1 and 2 only
B
3 and 4 only
C
1, 3 and 4 only
D
2 and 3 only

288

What is narrow money?

  1. M1
  2. M2
  3. M3
  4. M4

Select the correct code:

A
1 and 2 Only
B
1 Only
C
3 and 4 Only
D
2 and 4 Only

289

Which of the following is not true about monetary and credit policy of India?

A
The Annual Monetary and Credit Policy in India is announced in the month of April every year.
B
The mid-term review of the monetary and credit policy is done in the month of October every year.
C
RBI also does quarterly review of the monetary and credit policy.
D
RBI drafts and implements the monetary and credit policy in consultation with the Ministry of Finance.

290

The term ‘Food Credit’ in the context of RBI’s Credit/Monetary Policy implies:

A
Credit available with the people to purchase food.
B
Credit given by banks to fair price shops.
C
Credit given by banks to farmers for production of food grains.
D
Credit given by banks to Food Corporation of India for procurement of food grains.

291

Consider the following statements regarding Gold Monetisation Scheme-

  1. The main aim of the scheme is to encourage the people to deposit jewellery, bars or coins with banks so it can be refined to meet fresh demand and cit the need for imports.
  2. The Central government will pay 2.5% commission to bank that will make the scheme attractive for the banks.

Which of the statement given above is/are correct?

A
I Only
B
II Only
C
Both
D
None

292

Identify the true statements regarding the depreciation of the value:

  1. Depreciation of value is an accounting concept
  2. To adjust the regular damage of capital goods, value that is subtracted from the total investment is called depreciation
  3. Damages caused by the sudden destruction, natural disaster or misuse of capital are also included in depreciation.

Choose the correct statement/s from the choices given below:

A
1 and 2
B
Only 2
C
2 and 3
D
All of the above

293

Which of the following functions is performed by the currency in an economy?

A
As a means of exchange
B
As a unit of convenient accounting
C
Function of value accumulation
D
All of the above

294

Which of the following is called Currency Deposit Ratio?

A
Ratio of amount of cash a person holds and currency deposited in the bank
B
Ratio of amount of cash a person holds and accumulated goods in the form of precious metals
C
Both of the above
D
None of the above

295

The qualitative tool of RBI's monetary policy included:

  1. Rationing of credit
  2. Moral impact
  3. Open market activities

Codes:

A
1 and 2
B
1 and 3
C
2 and 3
D
1, 2 and 3

296

Which of the following functions are served by the money?

  1. It acts as a medium of exchange.
  2. It is a convenient unit of account.
  3. It stores value for individuals.

Select the correct code:

A
1 and 2 only
B
2 and 3 only
C
1 and 3 only
D
All of the above

297

What are the reasons for depreciation of currency in India?

  1. Crude oil import
  2. Growing Trade deficit
  3. High inflation

Select the correct code:

A
1 and 2
B
3 only
C
1 and 3
D
All of the above

298

With reference to Monetary Policy Framework Agreement announced between RBI and Union Government, which of the given statement is/are correct?

  1. RBI will publish a document every six month explaining sources of inflation and forecast of the inflation for subsequent six months.
  2. Any dispute regarding the interpretation and implementation of the agreement will be decided by Supreme court of India.
  3. The agreement mandates RBI to publish the operating procedures of monetary policy and changes in operating procedures that are used to achieve targets.

Select the correct answer using the code given below:

A
Only 3
B
1 and 3
C
1 and 2
D
1, 2 and 3

299

Match list 1 with List 2 and select the correct answer using the codes given below the lists:

List 1 List 2
A. Deflation 1. A continuous decrease in general price level.
B. Disinflation 2. Raising price deliberately to relieve depression.
C. Stagflation 3. Fall in prices without any decline in output.
D. Reflation 4. Recession accompanied by inflation.

Codes:

A B C D

A
1 3 4 2
B
1 3 2 4
C
3 1 4 2
D
3 1 2 4

300

Consider the following statements-

  1. Increasing amount of NPA leads to inflation because of excess demand of loan in the market.
  2. Inflation causes by excess supply of money in the market.
  3. To curtail the impact of inflation RBI raises Repo rate.

Which of the statement given aboveis /are correct?

A
II and III only
B
I and III only
C
I and II only
D
All